2022 AMC 12A Problems/Problem 18

Revision as of 21:04, 11 November 2022 by Stevenyiweichen (talk | contribs) (Created page with "==Problem== Let <math>T_k</math> be the transformation of the coordinate plane that first rotates the plane <math>k</math> degrees counter-clockwise around the origin and the...")
(diff) ← Older revision | Latest revision (diff) | Newer revision → (diff)

Problem

Let $T_k$ be the transformation of the coordinate plane that first rotates the plane $k$ degrees counter-clockwise around the origin and then reflects the plane across the $y$-axis. What is the least positive integer $n$ such that performing the sequence of transformations $T_1, T_2, T_3, \cdots, T_n$ returns the point $(1,0)$ back to itself?

Solution

https://youtu.be/QQrsKTErJn8

(Professor Chen Education Palace, www.professorchenedu.com)